[Rozgrzewka OM][MIX][Teoria liczb] Teoria liczb

Zadania z kółek matematycznych lub obozów przygotowujących do OM. Problemy z minionych olimpiad i konkursów matematycznych.
Regulamin forum
Wszystkie tematy znajdujące się w tym dziale powinny być tagowane tj. posiadać przedrostek postaci [Nierówności], [Planimetria], itp.. Temat może posiadać wiele różnych tagów. Nazwa tematu nie może składać się z samych tagów.
Awatar użytkownika
Vax
Użytkownik
Użytkownik
Posty: 2913
Rejestracja: 27 kwie 2010, o 22:07
Płeć: Mężczyzna
Lokalizacja: Biała Podlaska / Warszawa
Podziękował: 4 razy
Pomógł: 612 razy

[Rozgrzewka OM][MIX][Teoria liczb] Teoria liczb

Post autor: Vax »

A tak serio:
Ukryta treść:    
Znajdź wszystkie pary liczb całkowitych dodatnich \(\displaystyle{ (a,b)}\) dla których \(\displaystyle{ 7^a - 3^b \mid a^4+b^2}\)
ordyh
Użytkownik
Użytkownik
Posty: 255
Rejestracja: 6 paź 2009, o 18:04
Płeć: Mężczyzna
Pomógł: 66 razy

[Rozgrzewka OM][MIX][Teoria liczb] Teoria liczb

Post autor: ordyh »

Ukryta treść:    
Wykaż, że dla każdego \(\displaystyle{ n \in \mathbb{N}}\) można znaleźć trzy różne liczby \(\displaystyle{ a,b,c}\) większe od \(\displaystyle{ n^2}\) i mniejsze od \(\displaystyle{ n^2+n+3\sqrt{n}}\) takie, że \(\displaystyle{ a|bc}\).-- 10 sie 2013, o 14:45 --
hint:    
Awatar użytkownika
Vax
Użytkownik
Użytkownik
Posty: 2913
Rejestracja: 27 kwie 2010, o 22:07
Płeć: Mężczyzna
Lokalizacja: Biała Podlaska / Warszawa
Podziękował: 4 razy
Pomógł: 612 razy

[Rozgrzewka OM][MIX][Teoria liczb] Teoria liczb

Post autor: Vax »

Ukryta treść:    
Rozwiązać w \(\displaystyle{ \mathbb{Z}_+}\) równanie \(\displaystyle{ x!+y^3 = 18+z^3}\)
diana7
Użytkownik
Użytkownik
Posty: 78
Rejestracja: 17 lip 2012, o 20:38
Płeć: Kobieta
Lokalizacja: Bielsko-Biała
Pomógł: 13 razy

[Rozgrzewka OM][MIX][Teoria liczb] Teoria liczb

Post autor: diana7 »

Ukryta treść:    
Dane są \(\displaystyle{ a,b,n \in \mathbb{Z}_+}\), takie, że \(\displaystyle{ 2^n-1=ab}\), oraz \(\displaystyle{ k \in \mathbb{Z}_+}\), takie że \(\displaystyle{ ab+a-b-1 \equiv 2^k \pmod{2^{k+1}}}\). Pokazać, że \(\displaystyle{ k}\) jest parzyste.
kaszubki
Użytkownik
Użytkownik
Posty: 867
Rejestracja: 12 kwie 2008, o 13:35
Płeć: Mężczyzna
Podziękował: 6 razy
Pomógł: 78 razy

[Rozgrzewka OM][MIX][Teoria liczb] Teoria liczb

Post autor: kaszubki »

jest forma przed memo:    
Udowodnij, że \(\displaystyle{ \sum_{i=1}^{\frac{p-1}{4}} \lfloor \sqrt{ip} \rfloor = \frac{p^2-1}{12}}\) gdzie \(\displaystyle{ p}\) jest liczbą pierwszą taką, że w ciągu arytmetycznym \(\displaystyle{ p-1,2p-1,...}\) występuje co najmniej jeden kwadrat liczby naturalnej.
MadJack
Użytkownik
Użytkownik
Posty: 270
Rejestracja: 21 lis 2010, o 22:23
Płeć: Mężczyzna
Podziękował: 5 razy
Pomógł: 35 razy

[Rozgrzewka OM][MIX][Teoria liczb] Teoria liczb

Post autor: MadJack »

Jakaś wskazówka? Albo ktoś ma jakiś pomysł?
Awatar użytkownika
Ponewor
Moderator
Moderator
Posty: 2218
Rejestracja: 30 sty 2012, o 21:05
Płeć: Mężczyzna
Lokalizacja: Warszawa
Podziękował: 70 razy
Pomógł: 297 razy

[Rozgrzewka OM][MIX][Teoria liczb] Teoria liczb

Post autor: Ponewor »

Ukryta treść:    
Oildale
Użytkownik
Użytkownik
Posty: 59
Rejestracja: 29 gru 2012, o 23:06
Płeć: Mężczyzna
Lokalizacja: Pruszków
Pomógł: 3 razy

[Rozgrzewka OM][MIX][Teoria liczb] Teoria liczb

Post autor: Oildale »

Wskazówka: Pomieszać sumę i wykorzystać fakt, że \(\displaystyle{ -1}\) jest resztą kwadratową modulo \(\displaystyle{ p}\).
Awatar użytkownika
mol_ksiazkowy
Użytkownik
Użytkownik
Posty: 11360
Rejestracja: 9 maja 2006, o 12:35
Płeć: Mężczyzna
Lokalizacja: Kraków
Podziękował: 3153 razy
Pomógł: 747 razy

[Rozgrzewka OM][MIX][Teoria liczb] Teoria liczb

Post autor: mol_ksiazkowy »

Wyznaczyć wszystkie liczby pierwsze \(\displaystyle{ p, q}\) takie, że \(\displaystyle{ m^{3pq} \equiv m \pmod{ 3pq}}\) dla dowolnej liczby naturalnej \(\displaystyle{ m}\)
Ostatnio zmieniony 21 paź 2018, o 00:37 przez Jan Kraszewski, łącznie zmieniany 1 raz.
Powód: Poprawa wiadomości.
Awatar użytkownika
Premislav
Użytkownik
Użytkownik
Posty: 15687
Rejestracja: 17 sie 2012, o 13:12
Płeć: Mężczyzna
Lokalizacja: Warszawa
Podziękował: 195 razy
Pomógł: 5220 razy

Re: [Rozgrzewka OM][MIX][Teoria liczb] Teoria liczb

Post autor: Premislav »

Ale to już było i nie wróci więcej, i choć tyle się zdarzyło, to do przodu wciąż wyrywa głupie serce.
56706.htm
Awatar użytkownika
mol_ksiazkowy
Użytkownik
Użytkownik
Posty: 11360
Rejestracja: 9 maja 2006, o 12:35
Płeć: Mężczyzna
Lokalizacja: Kraków
Podziękował: 3153 razy
Pomógł: 747 razy

[Rozgrzewka OM][MIX][Teoria liczb] Teoria liczb

Post autor: mol_ksiazkowy »

Udowodnić, że jeśli \(\displaystyle{ n>1}\) i \(\displaystyle{ 2n-1}\) oraz \(\displaystyle{ 3n-2}\) są kwadratami liczb całkowitych, to \(\displaystyle{ 19n - 14}\) jest liczbą złożoną
Awatar użytkownika
kerajs
Użytkownik
Użytkownik
Posty: 8581
Rejestracja: 17 maja 2013, o 10:23
Płeć: Mężczyzna
Podziękował: 307 razy
Pomógł: 3349 razy

Re: [Rozgrzewka OM][MIX][Teoria liczb] Teoria liczb

Post autor: kerajs »

\(\displaystyle{ 19n-14=9(3n-2)-4(2n-1)=9a^2-4b^2=(3a-2b)(3a+2b)}\)
Awatar użytkownika
mol_ksiazkowy
Użytkownik
Użytkownik
Posty: 11360
Rejestracja: 9 maja 2006, o 12:35
Płeć: Mężczyzna
Lokalizacja: Kraków
Podziękował: 3153 razy
Pomógł: 747 razy

[Rozgrzewka OM][MIX][Teoria liczb] Teoria liczb

Post autor: mol_ksiazkowy »

oraz \(\displaystyle{ 3a-2b>1}\). Można przedstawić nastepne zadanie...
Awatar użytkownika
Premislav
Użytkownik
Użytkownik
Posty: 15687
Rejestracja: 17 sie 2012, o 13:12
Płeć: Mężczyzna
Lokalizacja: Warszawa
Podziękował: 195 razy
Pomógł: 5220 razy

Re: [Rozgrzewka OM][MIX][Teoria liczb] Teoria liczb

Post autor: Premislav »

Czy istnieje \(\displaystyle{ n\in \NN^+}\) takie, że \(\displaystyle{ \sin n}\) jest liczbą wymierną? A jak będzie z \(\displaystyle{ \sin n!}\)?
Awatar użytkownika
kerajs
Użytkownik
Użytkownik
Posty: 8581
Rejestracja: 17 maja 2013, o 10:23
Płeć: Mężczyzna
Podziękował: 307 razy
Pomógł: 3349 razy

Re: [Rozgrzewka OM][MIX][Teoria liczb] Teoria liczb

Post autor: kerajs »

Negatywna odpowiedź na oba pytania wynika z niealgebraiczności sinusa.
mol_ksiazkowy pisze: jeśli \(\displaystyle{ n>1}\) i \(\displaystyle{ 2n-1}\) oraz \(\displaystyle{ 3n-2}\) są kwadratami liczb całkowitych
Dla jakich n powyższe liczby będą kwadratami?

PS
Przykładowe n spełniające treść zadania: 7366416376406081 i 41.
ODPOWIEDZ